Vous êtes sur la page 1sur 30

TORTS OUTLINE Fall 07 Prof.

Walker

I.

Procedures at Trial
a.
b.
c.
d.
e.
f.

II.

Motion to Dismiss / Demurrer


Motion for Summary Judgment
Motion for Directed Verdict
Request for / Objection to Jury Instructions
Motion for JNOV
Motion for New Trial

Intentional Torts
a. BATTERY an actor is subject to liability to another for battery if (a) he acts intending to cause a harmful
or offensive contact with another person, or an imminent apprehension of such contact, and (b) a
harmful contact with the other person directly or indirectly results (Restatement 2d 13)
i.
An intentional, unconsented-to harmful or offensive contact with another; contact which is
offensive to a reasonable sense of personal dignity is offensive contact
ii.
Van Camp v. McAfoos woman was hit in the ankle by a 5yr old on a tricycle; court held that just
claiming injury is not enough for relief; must also prove intent or negligence
1. Child liability in most states child cannot escape tort liability simply because of
young age (but age can be considered in determining whether there was intent)
2. Common law rule parents are not vicariously liable for the torts of their children
a. Statutes may impose liability on parents in certain cases (ex. If childs tort
was committed wantonly or willfully), but not for every common-law
intentional tort, just those that fall within the statutory definitions giving rise
to liability
iii.
Defendant can commit a battery on plaintiffs body by making contact with an object that is
attached to it and practically identified with it (ex. Snatching a purse from plaintiffs hand)
iv.
Snyder v. Turk contact does not have to cause physical harm to be considered harmful or
offensive contact
v.
Leichtman v. WLW Jacor Communications court held that intentionally blowing smoke in
someones face could be battery b/c smoke was offensive contact, meaning disagreeable or
nauseating or painful b/c of outrage to taste and sensibilities or affronting insultingness
vi.
Establishing Intent (Garratt v. Dailey) substantial certainty test court held that intent is
established if it is proven that defendant knew with substantial certainty that his act would result
in the harmful contact or apprehension of such contact (otherwise, defendant did not have the
intention necessary for liability); it does not matter what his purpose was
1. Restatement 2d 8A intent denotes that the actor desires to cause
consequences of his act, or that he believes that the consequences are substantially
certain to result from it
2. Willful or Wanton conduct course of action which shows actual or deliberate intent
to harm, or which shows an utter indifference for personal safety or the safety of
others (it is a hybrid between acts considered negligent and behavior found to be
intentionally tortuous)
3. Intent is not about the act; its about the consequences of the act
vii.
Hall v. McBryde intent element can be satisfied even if actor did not intend harmful contact; it
is enough for intent to prove that he intended to cause apprehension of a harmful or offensive
contact (defendant could be liable to bystander neighbor b/c transfer of intent see
Restatement 2d 16(2)). Two types of transferred intent:
1. Tortfeasor intends a tort to one person, but commits a tort on another
2. Tortfeasor intends one tort, but accomplishes another
viii.
Polmatier v. Russ court held that defendant can be insane but still have the kind of intent
necessary to establish battery; it is not necessary for a defendants reasons and motives for
1

forming his intention to be rational in order for him to have the intent to invade the interests of
another. Reasoning for holding:
1. where case involves two innocent people, the one who caused the harm should bear
the loss of compensating the other
2. such liability must be enforced so that relatives of the insane person have a reason to
restrain them
3. deters other sane tortfeasors from feigning insanity to avoid liability
ix.
Single Intent vs. Dual Intent (White v. Muniz)
1. Single Intent tortfeasor intended a contact (contact must be such that a reasonable
person would find offensive or to which victim did not consent); tortfeasor need not
have intended the harm that actually resulted
2. Dual Intent tortfeasor intended to contact and intended the contact to be harmful
or offensive (tortfeasor must appreciate the offensiveness of her conduct)
b. ASSAULT an actor is subject to liability to another for assault if (a) he acts intending to cause a harmful
or offensive contact with another person, or an imminent apprehension of such a contact, and (b) the
other person is thereby put in such imminent apprehension (Restatement 2d 21)
i.
Any act of such a nature as to excite an apprehension of an imminent battery may constitute
assault. It is the right to be free from the apprehension of a battery which is protected by the
tort action which we call an assault.
1. The apprehension must be one which would normally be aroused in the mind of a
reasonable person
2. Imminent does not mean immediate; it means rather that there will be no significant
delay
ii.
Assault is a touching of the mind, as opposed to body, so damages which are recoverable for
assault are damages for mental trauma and distress (in addition to same kind of damages
recoverable for battery)
iii.
Words alone do not constitute assault must be coupled with acts or other circumstances
(Restatement 2d 31)
c. FALSE IMPRISONMENT an actor is subject to liability for false imprisonment if (a) he acts intending to
confine the other person within boundaries fixed by the actor, and (b) his act directly or indirectly results
in such a confinement, and (c) the other is conscious of the confinement or is harmed by it (Restatement
2d 35)
i.
False imprisonment occurs when a person confines another intentionally without lawful privilege
and against his consent within a limited area for any appreciable time, however short. The
plaintiff must have been aware of the confinement at the time or else suffered some actual harm
ii.
False imprisonment is a trespassory tort, so the plaintiff can recover damages even if she sustains
no actual harm
d. TORTS TO PROPERTY
i.
Trespass to Land Restatement 2d 158
1. Intentional entry upon land of another; may be accomplished by personal entry or by
causing an object to enter the land
2. The object of the intent need not be to trespass it is enough if defendant
intended to enter the land
3. Trespasser is liable for damages inflicted even if he never intended harm and could
not foresee it
ii.
Conversion of Chattels (Trover) intentional exercise of dominion or control over a chattel which
so seriously interferes with the right of another to control it that the actor may justly be required
to pay the other the full value of the chattel (Restatement 2d 222A)
1. No requirement that defendant be conscious of wrongdoing
2. Dominion matter of how serious the interference is, determined by:
a. Extent and duration of control
b. Defendants intent to assert a right to property
2

III.

c. Defendants good faith


d. The harm done
e. Expense or inconvenience caused
3. Usual remedy = damages measured by the value of the chattel at the time of
conversion
iii.
Trespass to Chattels intentionally dispossessing another of the chattel, or using or
intermeddling with a chattel in the possession of another (Restatement 217)
e. FORCIBLE HARMS AS CIVIL RIGHTS VIOLATIONS - 1983 Claim cause of action created by statute
(protects against deprivation of any rights, privileges, or immunities secured by the Constitution and
laws)
DEFENSES TO INTENTIONAL TORTS issues raised by defendant, which if defendant proves the
elements, is a complete defense to a claim of battery, assault, etc.; rebutting an element of the prima facie case
is not an affirmative defense
a. SELF-DEFENSE one is privileged to use reasonable force to defend against harmful or offensive bodily
contact and against confinement; privilege depends on apparent necessity of self-defense, not on actual
reality (ex. If defendant reasonably but mistakenly believes she is being attacked)
i.
Defendants privilege extends only so far as reasonably necessary to prevent harm; if the harm
threatened is not itself death or serious injury, then the defendant may not use force likely to
cause death or serious injury
1. Privilege only covers reasonable force
ii.
Defendant who is attacked is not required to retreat or otherwise avoid the need for self-defense
iii.
Provocation is not sufficient to raise the self-defense privilege
b. DEFENSE OF THIRD PERSONS general rule is that one may defend others on the same basis that he may
defend himself (see Restatement 2d 76)
c. ARREST AND DETENTION
i.
Great Atlantic & Pacific Tea Co. v. Paul
1. defendant (store) urged court to adopt Restatement rule (One who reasonably
believes that another has tortuously taken a chattel upon his premises, or has failed
to make due cash payment for a chattel purchased or services rendered there, is
privileged, without arresting the other, to detain him on the premises for the time
necessary for a reasonable investigation of the facts).
2. Court retains the common law rule that shopkeeper has privilege to detain against
his will any person he believes has tortuously taken his property (only applies to
prevention of theft or recapture of property, not to punishment); shopkeeper
exercises this privilege as his peril because if person detained does not in fact
unlawfully have any of shopkeepers property, shopkeeper is liable for false
imprisonment
d. DEFENSE & REPOSSESSION OF PROPERTY
i.
Katko v. Briney use of a loaded spring gun to protect property was held to be an unreasonable
use of force; court reasons that value of human life and limb outweighs property rights (property
owner is not privileged to use deadly force unless trespasser was endangering human life by his
act committing a violent felony)
ii.
Brown v. Martinez court held landowner liable for shooting boy trespassing to steal
watermelons; there was no evidence that landowner felt his safety was threatened, so shooting
his gun was an unreasonable use of force; there is no privilege to use any force calculated to
cause death or serious bodily injury where only the property is threatened
e. CONSENT
i.
In battery cases, consent or apparent consent conveys the idea that a touching is not offensive;
whether consent is considered an element of battery or an affirmative defense affects who has
the burden of proving consent
1. Person may not consent to touching in certain public situations (ie. Subway) but we
have to consider whether the individuals feelings reflect those of a reasonable
3

person, and also have to consider that others are also privileged to use the subway in
a reasonable way
ii.
Reavis v. Slominski consent is not effective if a person lacks capacity to give consent
1. Incapacity of an adult plaintiff renders her consent ineffective only if her condition
substantially impairs her capacity to understand and weigh the harm and risks of
harm against the benefits flowing from the proposed conduct
2. Plaintiffs incapacity does not render her consent ineffective unless the defendant
has knowledge of that incapacity
iii.
Scope of Consent doctors failure to operate within the limits of the patients consent may be
brought as battery actions when a doctor treats a patient without his consent, or when a
doctor exceeds the scope of a patients consent, that is battery (scope of consent is not limited to
medical cases)
iv.
Consent of Minors general rule is that minors may consent to a number of touching
appropriate to their age
v.
Plaintiff can revoke her consent at any time by communicating her revocation to the defendant
f. NECESSITY privilege not based on plaintiffs conduct
i.
Public Necessity one is privileged to enter land in the possession of another if it is, or if the
actor reasonably believes it to be, necessary for the purpose of averting an imminent public
disaster (Restatement 2d 196)
1. Surocco v. Geary plaintiffs could not recover for loss of the goods in their home
when the house was blown up to try to stop an advancing fire; court held that the
private rights of the individual yield to the considerations of general convenience,
and the interests of society
2. Wegner v. Milwaukee Mutual state law said that private property could not be
taken without just compensation; court rejects the doctrine of public necessity
a. Policy reasoning innocnet private party should not have to bear the entire
risk of loss for the good of the public
ii.
Private Necessity one is privileged to enter or remain on land in the possession of another if it
is or reasonably appears to be necessary to prevent serious harm to: (a) the actor or his land or
chattels, or (b) the other or a third person or the land or chattels of either, unless the actor
knows or has reason to know that the one for whose benefit he enters is unwilling that he shall
take such action (Restatement 2d 197)
1. Vincent v. Lake Erie Transportation court held that defendant was prudent in
securing his ship to the dock, but that plaintiff still deserves compensation
(preserved ship at docks expense)
g. TYPES OF PRIVILEGE
i.
Absolute privilege ex. Cant sue a judge for false imprisonment if they incorrectly lock you up
ii.
Qualified privilege reasonable under the circumstances
iii.
Limited privilege such as private necessity

IV.

NEGLIGENCE PRIMA FACIE CASE


a. ELEMENT 1 Duty of Care
i.
Reasonable Person Standard to exercise the care that would be exercised by a reasonable and
prudent person under the same or similar circumstances to avoid or minimize risks of harm to
others (Restatement 283)
1. Stewart v. Motts court held that there does not exist a higher standard of
extraordinary care when dealing with dangerous instrumentalities; there is only the
reasonable person standard - the care employed by a reasonable man must be
proportionate to the danger of the activity
2. Wilson v. Sibert court held it appropriate to include a sudden emergency jury
instruction; when person is faced with a sudden emergency, that factor may be
taken into account to determine if his actions were those of a reasonable person
(Restatement 296) does not change the standard of care
4

ii.

a. How a reasonably prudent person acts can be different in emergency


situations, but not if they are responsible for the emergency
b. Courts split on whether emergency instructions are prejudicial
3. Robinson v. Lindsay court held defendant (13yr old) to an adult standard of care
because the activity he engaged in (snowmobiling) was inherently dangerous (this
case represents the exception to the general child standard of care rule)
a. Child Standard of Care: reasonable person of like age, intelligence, and
experience under like circumstances (Restatement 283A)
b. Exception: when a child engages in an activity which is normally undertaken
only by adults, he may be held to an adult standard of care
i. Policy reasoning: to discourage children from engaging in activities
that require an adult level of care and competence
4. Creasy v. Rusk mental disability does not excuse a person from liability for conduct
that does not conform to the standard of a reasonable man under like circumstances
(Restatement 283B); mental disability is not considered a part of actors
circumstances
a. Policy reasons:
i. Allocates losses between 2 innocent parties to the one who caused
the loss
ii. Incentive for those responsible for disabled persons to prevent harm
and restrain those who are potentially dangerous
iii. Removes inducement for alleged tortfeasors to feign mental
disability
iv. Removes difficulty of courts and jurors to determine what is mental
disability and what is mere temperament
b. Despite mental disability not being an excuse for liability, court held
defendant not liable because he owed no duty of care to his caregiver (RULE:
duty of care is a one-way street, from caretaker to patient)
5. Physical illness or disability is taken into account in determining standard of care;
become part of the circumstances under which a reasonable person must act
(Restatement 283C)
6. If a person possesses more than the minimum required attention, knowledge,
intelligence, etc., he is required to exercise the superior qualities that he has in a
manner reasonable under the circumstances (Restatement 289)
7. Roman v. Estate of Gobbo man suffered heart attack while driving and killed other
motorists; defendant was found not liable he did not act unreasonably (sudden
heart attack was unforeseeable), and he had an affirmative defense for statutory
violation (sudden medical emergency incapacitation)
Negligence Per Se court adopts a legislative enactment or an administrative regulation as a
replacement rule (to the reasonable person standard) that states the standard of care under
specified circumstances
1. Not the same as a statutory cause of action (ex. 1983); the Legislature may enact a
penal statute that does not explicitly provide a civil remedy, and the courts may then
derive a civil legal duty from the penal statute (courts do not have to adopt statutory
standard; if they dont, then we return to the default reasonable person standard)
2. To declare conduct negligent per se is to render conduct negligent as a matter of law
not the same as a finding of liability per se (plaintiff must still establish cause in
fact, proximate cause, and damages)
3. Rains v. Bend of the River two threshold questions (if these two are satisfied, then
courts are inclined to adopt the statute as a common law cause of action):
a. Does plaintiff belong to the class of persons the statute was designed to
protect?
5

b. Is plaintiffs injury of the type the statute was designed to prevent?


4. Impson v. Structural Metals, Inc. defendants excuses were not legally acceptable
for his violation of no passing within 100 feet of an intersection; so he was liable for
negligence per se
a. Restatement 288A an excused violation of a legislative enactment is not
negligence (the section sets forth five categories of excuses)
b. ELEMENT 2 Breach of Duty has two parts: (1) convincing the jury that defendant engaged in the
conduct, and that (2) the conduct was negligent
i.
Assessing Reasonable Care by Assessing Risks and Costs
1. Indiana Consolidated Insurance Co. v. Mathew court held that defendant did not
breach the reasonable person duty of care; he fueled the lawnmower properly,
started it up in the garage (just as a reasonable person wouldve done risk of the
engine catching fire was not foreseeable), and then acted prudently by protecting his
own safety rather than trying to save property from harm
2. Stinnett v. Buchele defendant (employer) was found not liable of negligence;
employer is not obligated to provide absolutely safe work environment; employer
was not negligent for failing to warn plaintiff of the danger of falling off the roof
because it is such an obvious danger; employer was reasonable in relying on
plaintiffs expertise (so if plaintiff required safety harnesses, he shouldve asked)
a. Policy argument: other things being equal, the party in the best position to
weigh the risks and benefits should be the party to bear the costs of an
accident
3. Bernier v. Boston Edison Co. the defendant power company was liable for negligent
design and continued use of lamp poles because vehicular collisions were
foreseeable and thus created an unreasonable risk (the company was aware of the
risk of the poles crumbling upon collision, and couldve made them safer with just
minor alterations)
4. United States v. Carroll Towing Co. risk-utility weighing
a. Learned Hand Formula liability depends upon whether the burden to
avoid the harm is less than the gravity of the potential harm multiplied by
the probability that it will occur (B < PL)
i. B = Burden of taking precautions or avoiding the potential harm
ii. P = Probability that harm will occur
iii. L = Gravity of the potential harm
b. If you create a risk in society and the avoidance of that risk would not have
been burdensome, then thats negligence
c. See Restatement 291-293
ii.
Assessing Responsibility When More Than One Person Is Negligent
1. Comparative Fault plaintiffs recovery is reduced to reflect her fault (not ordinarily
in intentional torts, generally in negligence and strict liability cases)
2. Apportionment among defendants
a. Joint and Several Liability the plaintiff can enforce her tort claim against
either tortfeasor (she can obtain a judgment against both, but she cannot
collect more than her full damages)
b. Contribution if one defendant pays the entire judgment, he can obtain
contribution from the other defendant so that his payment is proportional to
his fault
iii.
Proving Conduct
1. Santiago v. First Student, Inc. student hit her face when school bus collided with
another car in an intersection; court granted summary judgment for defendant
because plaintiff could offer no details regarding defendants negligent conduct (the
fact that plaintiffs case may be difficult to prove does not relieve her of the burden
6

iv.

of presenting sufficient evidence to demonstrate the existence of a material question


of fact)
2. Upchurch v. Rotenberry plaintiff motioned for JNOV; appellate court affirmed jury
verdict concluding that there was sufficient evidence in dispute (not an
overwhelming weight of evidence on one side or the other), and so reasonable jury
could reach different conclusions
a. It is not up to the appellate court to determine the credibility of witnesses or
sort through facts
3. Types of Evidence:
a. Direct if the testimony of the witness is credible, then the issue of fact is
proven
b. Circumstantial even if the testimony is credible, it still requires some
inferences by the trier of fact to prove the issue theyre concerned about;
evidence of one fact permits an inference of another fact
4. Types of Knowledge:
a. Actual I have subjective knowledge of some state of affairs in the
environment; I am aware of the risk
b. Constructive a reasonably prudent person wouldve known in those
circumstances; negligence consists of not knowing
5. Witnesses Opinions:
a. Non-expert not usually permitted to give opinions on ultimate issues that
are reserved for jury decision (ex. In my opinion, the defendant was
negligent)
b. Expert usually permitted to give expert opinion or conclusions within the
field of their expertise, provided the testimony is likely to be helpful to the
jury on an issue in the case; may be excluded if jury could readily determine
the issue by interpreting the facts for themselves
c. Problems w/expert opinions:
i. Experts often differ
ii. Witness may not in fact be an expert with respect to his particular
testimony
iii. May overwhelm the jury
Evaluating Conduct
1. Thoma v. Cracker Barrel trial court granted summary judgment, which appellate
court reversed because many different reasonable inferences could be drawn from
the evidence, and so it shouldve been left up to the jury to decide whether a
preponderance of the evidence supported the inferences suggested by plaintiff
a. 3 common theories of liability in restaurant slip & fall
i. Defendant created the dangerous situation
ii. Defendant did not directly create the condition but discovered or
should have discovered it and should have taken precautions to
prevent injury
iii. Defendants mode of business operations made it all too likely that
others would create a dangerous condition
b. Chief method for proving that defendant shouldve discovered the
dangerous condition: showing that the substance had been there for a
relatively long time
2. Wal-Mart Stores v. Wright the subjective standard of care set forth in a companys
safety manual cannot replace the objective reasonable person standard; the facts of
the case should be judged by the legal standard, not by the standards the defendant
sets for itself
7

v.

a. Custom jury may consider customary industry practices to determine


whether defendant breached the reasonable person standard of care, but it
does not become the standard
i. This differs from per se negligence, which changes the duty of care
(jury instruction on element 1)
ii. Custom establishes what defendant shouldve known; evidence of
foreseeability of harm, unreasonableness of risk, feasibility of safety
precaution, etc.
iii. Custom aids the jury in comparing the defendants conduct with the
required norm of reasonable prudence
Res Ipsa Loquitur a judicially created and recognized set of conditions about which the jury is
instructed (if there is sufficient evidence, which may prove by circumstantial evidence that the
defendant engaged in conduct that was negligent); a way for plaintiff to put together
circumstantial evidence to get to the jury in the absence of any evidence of affirmative conduct
1. Designed especially for circumstances in which the plaintiff has no reasonable way of
producing evidence about any specific conduct; plaintiffs best evidence that
defendant engaged in negligent conduct is the nature of accident itself and the fact
that it happened (ex. Byrne v. Boadle barrel rolling out of the warehouse case)
2. The application of res ipsa means that, on element 2, the plaintiff will survive a
motion for directed verdict and get to the jury, which can then decide the case either
way
3. Traditional Res Ipsa Conditions (Eaton v. Eaton)
a. The accident was of a type that ordinarily does not happen unless someone
was negligent (making an inference to negligence based on the type of
accident)
b. The instrumentality or agent that caused this accident was under the
exclusive control of this defendant
c. The victim did not cause or contribute to this particular accident
4. Restatement 2d 328D Res Ipsa Conditions (Valley Properties)
a. The event is of a kind which ordinarily does not occur in the absence of
negligence
b. Other responsible causes [of this particular accident], including the conduct
of the plaintiff and third persons, are sufficiently eliminated by the evidence
c. (the indicated negligence is within the scope of the defendants duty to the
plaintiff)
5. Permissive Inference States jury instructions state that if they find the conditions
to be true, they may find that the defendant engaged in negligence (an inference of
negligence is permitted, not required)
a. Even if defendant introduces no evidence at all, the jury may reject the
inference and bring in a verdict for the defendant
b. Defendant does not have burden of production (but any sane defendant will
put on evidence to persuade the jury but theyre not required, just an
incentive to produce evidence)
c. Res Ipsa decided as a matter of law - court may grant directed verdict for
plaintiff if court finds that no reasonable jury could not find that all 3
conditions of res ipsa are met; but jury must still then decide element 2 of
the prima facie case (since in permissive inference, if the conditions of res
ipsa are met, the jury may find that there was negligence)
6. Presumptive Inference States (Burden Shifts) minority of jurisdictions; burden
shifts to the defendant if plaintiff proves the conditions of res ipsa to be true, either
by jury verdict or as a matter of law (this is not an affirmative defense)
8

7.

8.

9.

10.

a. Burden of producing evidence judge decides if this burden has been met;
you have to produce evidence for the record in the court case (if not,
directed verdict)
i. If plaintiff proves 3 conditions and defendant does not produce
evidence, then plaintiff is entitled to res ipsa as a matter of law and
jury must find element 2 of prima facie case in plaintiffs favor
b. Burden of persuading the factfinder jury decides if burden has been met by
the preponderance of the evidence (unless some issue of fact is decidable as
a matter of law without the jury)
c. What if the conditions of res ipsa are not decidable as a matter of law and
must go to the jury? Then until the jury comes back with its verdict on the
conditions, we dont know if theyre all true, which means we dont know if
the burden shifted or not (in that case, we handle the possibilities in the jury
instructions)
Questions to keep in mind concerning any burden shift:
a. Which burden shifts production only, or production and persuasion?
b. When does it shift (under what conditions)?
c. What are the jury instructions if it shifts?
JURY INSTRUCTIONS
a. Permissive Inference if jury finds plaintiff has proven all conditions of res
ipsa, they may find that defendant engaged in negligent conduct
b. Presumptive Inference w/production only shift: if plaintiff proves (by a
preponderance of the evidence) that the conditions of res ipsa are true,
then:
i. If defendant has produced sufficient evidence, then you may find
element 2 for the plaintiff
ii. If defendant has not produced sufficient evidence, then you must
find element 2 for the plaintiff
c. Presumptive Inference w/production & persuasion shift: if plaintiff proves
(by a preponderance of the evidence) that the conditions of res ipsa are true,
then:
i. If defendant has produced sufficient evidence, then you must find
element 2 for the plaintiff unless the defendant proves that he did
not engage in negligent conduct
ii. If defendant has not produced sufficient evidence, then you must
find element 2 for the plaintiff
Giles v. City of New Haven (liberal formulation of the control requirement) - elevator
operator suffered personal injuries when elevator chain broke; defendant argued
that conditions 2 and 3 of res ipsa were missing (conceded on condition 1 because it
would be bad for business to say that such accidents do occur in the absence of
negligence)
a. Court held that plaintiffs use of the instrument does not in and of itself bar
plaintiff from applying res ipsa; exclusive control is just one way to prove
defendants responsibility; it is enough that the defendant is under a duty
which he cannot delegate to another (see comment g to 328D of
Restatement)
b. Trial court must allow jury to draw inference of negligence if there is
sufficient evidence for a reasonable jury to find that defendant was more
likely responsible for the accident than someone else defendant did not
have to have exclusive control
Warren v. Jeffries car began rolling backward and ran over little boy when he tried
to jump out of the car; court held res ipsa inapplicable because plaintiff failed to
9

provide sufficient evidence (plaintiff only put forth theories of negligent conduct, but
no evidence to support them)
11. Widmyer v. Southwest Skyways, Inc. plane crash that killed all on board; court
held that:
a. res ipsa instruction shouldve been given because although plaintiffs expert
witness provided one possible explanation for the crash, the testimony was
based on circumstantial evidence and did not provide a complete factual
explanation of the accident (When evidence presents a complete explanation
for the accident, courts refuse to apply res ipsa)
b. superior knowledge on the part of the plaintiff precludes a res ipsa
instruction, but in this case the parties were equally ignorant of the cause of
the crash (and defendant had superior knowledge of the plane, its
maintenance, etc.)
c. in order to deny a res ipsa instruction, there must be some evidence that
plaintiff contributed to the accident (in a case where there is no such
evidence, the court presumes there was no contribution)
d. plaintiff is not required to negate the possibility of causation by weather (air
crashes do not normally occur absent negligence, even in inclement
weather)
12. If common knowledge would not allow jury to determine that the accident would
not occur absent negligence, plaintiff may present expert testimony if defendant
provides conflicting expert testimony, this does not rule out res ipsa
13. Newing v. Cheatham (Production Shift) private plane crash; plaintiffs had 3
theories of negligent conduct (negligently letting plane run out of fuel, negligence
per se in violating federal air regulations, and res ipsa loquitur); court decided
liability as a matter of law (not just on element 2, but on the issue of liability made
judgment on defenses as well) jury simply deliberated on damages
a. Court held that plaintiff had established 3 elements of res ipsa and that
defendant had not met its burden (failed to produce sufficient evidence of
defendants non-negligence), so granted plaintiffs motion for directed
verdict
i. Defendant brought up possibility of mechanical failure, had
testimony that the accident inspection was flawed, expert testimony
said didnt think that plane run out of fuel but this was insufficient
because defendant must produce some evidence of what caused the
accident if it wasnt the defendants negligence
ii. theories wont do it in a production shift state; you need
affirmative evidence
b. Condition 1 established by witness testimony (Rojo and Leon testified that
there was no evidence of a collision, etc)
c. Condition 2 established by FAA regulations (Cheatham had responsibility
for all aspects of control over the plane - this is not per se negligence; they
are not using the regulation as the standard of care; the regulation in this
case is about who is responsible for the operation of an airplane, not how to
operate the airplane)
d. Condition 3 established by preponderance of the evidence (Rojo and Leon
testified that body of Newing was found in the backseat of the plane at the
crash scene and court presumes that passenger did not contribute to the
accident, such as court did in Widmeyer it would be pure speculation to
think that Newing contributed to the crash)
c. ELEMENT 3 Actual Harm
10

i.

Plaintiff must suffer legally cognizable harm; nominal damages are not available in a negligence
action where no actual injury is proved (Restatement 907)
d. ELEMENT 4 Cause In Fact
i.
But-For Causation: but-for defendants negligent conduct, the plaintiffs injury would not have
happened
ii.
Salinetro v. Nystrom (Single Defendant Scenario) - Plaintiff was in car accident and subsequently
had an x-ray done; doctor (defendant) who did the x-ray did not ask if she was pregnant or not
(defendant herself did not know or even have reason to suspect); plaintiff went to gynecologist,
who advised her to have an abortion; the fetus was dead at the time of the abortion; plaintiff
filed lawsuit for medical malpractice
1. Plaintiffs theory of negligent conduct: negligently failing to ask if she was pregnant
before taking the x-ray
2. Defendant motioned for directed verdict, alleging that plaintiff had failed to establish
but-for causation; defendant argued that even if plaintiff was asked if she was
pregnant she would have said no b/c 6 days before shed been told by her
gynecologist that she was not pregnant (and she commonly had an irregular period),
so she would have had the x-ray anyway
3. Directed verdict was granted because no reasonable jury could find that failure to
ask caused the injury
iii.
Typical jury instruction for cause in fact: a proximate cause of the injury is a cause (there can be
multiple but-for causes) which in natural and continuous sequence (really element 5) produces
the injury and without which the injury would not have occurred (the but-for element) jury
instructions typically combine elements 4 and 5
iv.
You can sue multiple defendants for the same injury you can prove proximate cause against
multiple defendants (just because one defendant is a but-for cause does not mean others are
not)
1. Single, indivisible injury two defendants, each is a but-for cause
2. Concert of Action all you have to prove is that one defendant is a but-for cause,
and both are liable
v.
You can sue multiple defendants for two distinct injuries ex. one defendant injures plaintiffs
right arm, other defendant injures plaintiffs left arm in such a case, each defendant only pays
for the injury they caused
vi.
Landers v. East Texas Salt Water Disposal (Single Indivisible Injury) fish kill case; both
defendants caused saltwater to flow into the plaintiffs lake, killing his fish and causing other
damages
1. Defendants argued that there was insufficient evidence of concert of action (each
defendant was acting on its own), so the case must be treated as a multiple
defendant scenario, and since plaintiff cant prove which defendant caused what
damage, the plaintiff must lose the case
2. Court adopts a new rule to allow plaintiff get past summary judgment: the tortious
acts joined to produce an indivisible injury, and so all of the wrongdoers will be held
jointly and severally liable (and plaintiff may proceed to judgment against any one
separately or against all in one suit)
3. In theory the injury is divisible, but in practice indivisible; we let the defendants sort
out the injury among themselves; unless one of the defendants can prove they
caused no injury, they are partially liable (apportionment of the damages)
vii.
Anderson v. Minneapolis (Substantial Factor Test) two fires case; plaintiff alleged that his
property was burned by a fire caused by an engine of defendants train; defendant argued that
damage to plaintiffs property wouldve happened anyway because there was another fire (that
defendant didnt cause) approaching the property

11

viii.

ix.

x.

1. Defendant wanted regular but-for causation jury instruction, but court affirms the
alternative instruction given by trial court: a legal cause of injury is a cause which is
a substantial factor in bringing about the injury
2. Both the defendants fire and the other fire were independently sufficient causes of
the plaintiffs damages; where there is evidence sufficient for a jury to find that there
were two independent sufficient factors in the causation, then the jury should get a
substantial factor instruction
3. Policy reason for this alternative test: it would be unfair to plaintiff to require but-for
test; plaintiff would not be compensated simply because of the coincidence of the
second fire
Restatement
1. 430 Legal Cause (combines elements 4 and 5)
2. 431 differentiates between cause in fact and proximate cause
3. 432 breaks down cause in fact into but-for cause and substantial factor cause
Dillon v. Twin State Gas & Electric Co (Timing or Pre-existing Condition) - Decedent (14 yr old
boy) was electrocuted by an uninsulated wire running by a bridge; decedent lost his balance
while climbing the bridge and grabbed hold of the wire; wrongful death claim was brought by his
family
1. Family is suing for losses they suffered as a result of the death; pecuniary losses
what the person would have earned minus their personal expenses, the net earnings
are what the family is suing for (so a 14 yr olds contribution to the family income is
considered)
2. Defendant motioned for directed verdict on element 4, arguing that plaintiff failed to
establish but-for causation
a. Defense claims that decedent would have died anyway so they are not a butfor cause (regardless of plaintiffs claim of negligent conduct we dont
know exactly what it is could be design of the uninsulated wires,
maintenance of wires, lack of warning signs, placement of the wires, etc.)
b. Defendant argues that decedent would have died anyway even if wire did
not electrocute him; even if the fall had incapacitated him, his earning
capacity wouldve been severely limited
c. Plaintiff argues that we dont know what wouldve happened had the boy
fallen (so if the theory of negligent conduct is that the wires were not
insulated, plaintiff should be able to get to the jury)
d. Court denied the motion for directed verdict (court needs a jury to decide if
boy wouldve died from the fall without the electrocution, and had he fallen
if he wouldve still contributed to the family with some useful work)
Summers v. Tice (Burden Shift) 2 defendants, one of them caused the injury, and both of them
are defendants, and plaintiff could not reasonably prove the case against one or the other;
plaintiff and defendants went quail hunting; both defendants shot at the quail, in the direction of
the plaintiff; one shot hit plaintiffs eye
1. Court treats the eye injury as the bulk of the damages there can be only one butfor cause, but problem is both defendants engaged in negligent conduct and we
dont know whose gun the bullet came from
2. If there was concert of action, then it wouldnt matter who fired the bullet;
defendant argues that theres insufficient evidence for a reasonable factfinder to find
evidence of concert of action (the hunting itself cant be the conspiracy)
3. Not a case of joint & several liability (where both defendants are liable and the
question is are they liable for the whole injury) in this case the question is whether
either defendant is liable at all
a. Each defendant moves for directed verdict on element 4 theres
insufficient evidence to find that but-for the negligence of defendant,
12

plaintiffs eye would not be injured (sound argument, so plaintiff must argue
for new rules)
4. Plaintiffs argument for new rules:
a. Traditional rules are unfair because they let the defendants off the hook
even though both were negligent but defendant argues that it would be
unfair to make him pay for something he did not do (in this case we definitely
know that one of the defendants didnt do it wed be making an innocent
person pay)
5. Rule of Summers:
a. If each of the defendants is negligent
b. If only one of them caused the injury
c. If all possible tortfeasors are before the court, AND
d. If plaintiff cannot reasonably prove which defendant caused the injury, THEN
e. Burden of production AND persuasion shifts to defendants they have to
absolve themselves
6. Jury Instruction for Summers case:
a. If the plaintiff persuades you by a preponderance of the evidence that the
following conditions are true, that each defendant as negligent, and you find
that one of them caused the injury, but not both, and the plaintiff cannot
reasonably prove which one was the cause if you find those things to be
true, then you must find for the plaintiff against each defendant on element
4, unless one of the defendants prove to you they were not the cause
xi.
Lord v. Lovett (Pre-existing Injury / Lost Chance case) plaintiff suffered a broken neck in a car
accident; plaintiff claims that negligent misdiagnosis by defendants caused her to suffer paralysis
1. These types of cases are problematic because the pre-existing injury most of the
time ends up in paralysis anyway; plaintiff may have had only a 30% chance of
recovery anyway, but the defendants actions took away that chance and this is the
purpose of the malpractice lawsuit, to recover damages for the loss of that chance
2. Three traditional elements bar plaintiffs from recovering in these cases:
a. Standard of proof (preponderance of evidence)
b. But-for causation
c. Nature of plaintiffs compensable injury
3. To allow plaintiffs to recover, court didnt want to change but-for causation (and
cant really lower the standard of proof in civil case the standard is already low) so
they changed the rule on what counts as a compensable injury, recognizing lost
opportunity the damages awarded would be the value of the lost chance at
recovery
4. HOLDING: a plaintiff may recover for a loss of opportunity injury in med-mal cases
when the defendants alleged negligence aggravates the plaintiffs preexisting injury
such that it deprives the plaintiff of a substantially better outcome (problem is that in
none of these types of cases do we ever know for sure that the negligence
aggravated the condition)
5. This would not work as a substantial factor case b/c there are not 2 independently
sufficient causes (the defendants malpractice itself could not have caused paralysis
in the absence of the car accident)
6. Policy Argument: inherent in the doctor-patient relationship is a duty to give the
patient the best shot at recovery (and so courts should recognize as a tort when
doctors take away that chance)
e. ELEMENT 5 Proximate Cause
i.
The cause in fact chain (the link established in element 4) must be proximate to the injury; butfor causal chain could theoretically go back indefinitely, so there must be a line drawn in the
causal chain so that earlier but-for causes in the chain are not legally liable for negligence
13

ii.

iii.
iv.

v.

vi.

vii.

Medcalf v. Washington Heights Condo faulty intercom/buzzer system; plaintiff was attacked
while waiting outside for her friend to let her in the building; plaintiff claimed that if defendant
had maintained the intercom service, the injury wouldnt have happened (but for the negligent
maintenance of the door-release system, she would not have been standing outside to be
attacked)
1. Defendant motioned for directed verdict on element 5 - that the injury was not
reasonably foreseeable; saying that even if the jury found maintenance of the doorrelease to be negligent, and but-for the malfunction injury would not have
happened, as a matter of law element 5 has to be decided against the plaintiff b/c
its not a proximate cause in the causal chain
2. HOLDING: the risk of attack was not within the scope of the risk (was not
foreseeable) that made the conduct negligent, so court decides proximate cause as a
matter of law (plaintiff failed to establish element 5)
3. One could argue that jury should decide what risks are foreseeable
Restatement 435 the fact that harm was not foreseeable does not bar defendant of liability
(if defendants conduct is a substantial factor in bringing about the harm)
Palsgraf v. LIRR man rushed to get on train w/a package of fireworks under his arm; railroad
employee pushed him from behind to help him get on and the package fell to the tracks;
fireworks exploded causing scales at the other end of the platform to fall and injure the plaintiff
1. Cardozos argument: this case is all about foreseeability of risk and element 2; the
fact that weird sequence of unforeseeable events unfolded does not mean the
actions of the RR employee was negligent towards her; Mrs. Palsgraf was not in the
zone of danger of the pushing; case has to do with the scope of the duty, to whom is
a duty owed
2. Andrews argument: Andrews is in a different position than Cardozo b/c Andrews is
defending a jury verdict (saying this is a jury question hes not saying this is
obviously proximate), hes saying element 5 is not decidable as a matter of law, but
instead is a jury question
Integrating Palsgraf and Medcalf: defendant is liable only for harms within the scope of the risks
he negligently created specifically, defendant is liable only (a) for types of injuries risked by his
negligence and (b) to classes of persons risked by his negligence (these two elements are what
Andrews dissent argues to expand)
1. The defendant is not liable unless a reasonable person should have foreseen injuries
of the same general type that occurred and the general class of persons who would
suffer them
Rules for element 5 are very difficult to determine (you have to know how to make analogies to
the cases), but the way a court will decide on element 5 is very hard to predict
1. lawyers job is to make the best argument, regardless of the fact that judge may
disregard the argument
Rescue Cases
1. rescue is foreseeable as a matter of law and therefore proximate; the rescue is not a
superseding circumstance that alleviates the negligent party of liability (in these
cases unforeseeability is not a defense); danger invites rescue
2. the rescuer can recover from the defendant whose negligence prompts the rescue
(the rule includes cases in which the defendant negligently endangers himself and
the plaintiff is injured in attempting the rescue ex. Ouellette v. Carde, pg. 284 - the
rescuer was the plaintiff injured in the rescue attempt, and the court refused to
apply mere contributory negligence against the rescuer)
3. All the cases involving accidents during transportation to the hospital are also rescue
cases; ex. Anaya v. Superior Court pg. 266 helicopter crash case
a. Defendant is not relieved of liability because of the intervening act of a third
person if such act was reasonably foreseeable at the time of the original
14

viii.

ix.

x.

negligent conduct (rescue is foreseeable) the original tortfeasor is liable for


injuries or death suffered during transportation of the victim to a medical
facility
4. Negligent rescuers can also be liable, along with the original tortfeasor the
negligent rescue does not act as a superseding cause relieving original tortfeasor of
liability
a. The victim can sue the rescuer for negligence, but the negligent rescuer
should only pay for the additional harm caused by the rescue and not all of
the harm caused by the original tortfeasor (may be difficult to divide the
harm)
b. One who sees a person in imminent danger caused by the negligence of
another cannot be charged with contributory negligence unless the rescuer
acted recklessly (in this case the rescuer must be the plaintiff)
5. Restatement 457 restatement recognizes rescue cases as a bright-line rule
Thin Skull Cases
1. Defendant takes the plaintiff as he finds him
2. If you negligently injure someone, the peculiar susceptibility of the plaintiff does not
limit liability (unforeseeability of the magnitude of the harm does not matter)
3. Restatement 461
Hughes v. Lord Advocate post office employees left an unguarded manhole open, surrounded
by kerosene lanterns; boys were playing and knocked a lantern into the hole, causing an
explosion; one of the boys fell in and was badly burned
1. Court held that the unexpected explosion did not break the causal chain; the risk of
injury in leaving kerosene lanterns unattended was foreseeable (burns) so the fact
that the manner of occurrence was unusual was not a defense
2. RULE: the precise manner in which an injury occurs does not have to be foreseeable,
just the precise injury
3. Restatement 435(1)
Derdiarian v. Felix Contracting Corp Derdiarian was hit by a car on a construction site, thrown
into the air, and badly burned by hot liquid enamel
1. Con Ed hired Felix. Felix hired Bayside. Derdiarian works for Bayside. Dickens is the
driver that had the seizure. Felix told Derdiarian where to set up.
2. Derdiarian sues Felix, Dickens and Con Ed all 3 were found liable by jury
3. Plaintiff alleges Felix failed to put up proper barriers, put up a sign, or have two
flagmen present
a. Defendant argues that even if it was negligent not to have a stronger barrier,
and but-for the lack of a stronger barrier the accident wouldnt have
happened, Dickens broke the causal chain -- he was a superseding cause
(claim that the responsibility in the causal chain of Felix stopped because of
the conduct of Dickens)
4. Superseding Cause intervening act that breaks the chain of responsibility;
Restatement 440; intervening act is not necessarily a superseding cause; it
depends on whether the act is foreseeable if the act is extraordinary, not
foreseeable, or independent of the defendants conduct, it could be a superseding
cause (Restatement section 442)
a. An intervening act which falls squarely within the scope of the original risk
will not supersede defendants responsibility
5. Plaintiffs Counter-argument: a car coming through the barrier was foreseeable (like
Hughes manner of occurrence case), all that matters is the risk that made Felixs
conduct negligent is what occurred (why the vehicle went through the barrier is
irrelevant)
15

xi.

xii.
xiii.

V.

Sheehan v. City of New York Novak sued both drivers of bus and garbage truck, as well as
owners of both vehicles, for injuries sustained when the garbage truck hit the bus; bus had not
pulled over to the curb to let off passengers as its supposed to do
1. Sanitation truck argues that but-for the bus negligence in failing to pull over to the
curb as they were required to do (negligence per se) the accident would not have
occurred; superseding cause
2. Counter-argument: brakes of the truck failed and that a superseding cause; and even
if bus had a right to be in the designated lane, they also had a right to be in the
traveling lane (and even if bus had pulled over to let off passengers, and then pulled
back into the traveling lane, bus wouldve still been in a path of the truck)
3. The risk that made the failure to pull to the curb negligent (risk to passengers getting
on and off) was not the risk that occurred (bus being rear ended), so the bus
negligence is not a superseding cause in the trucks negligent maintenance of its
brakes (the accident happened to involve a bus, but it couldnt been a car, or a
person on a bike, etc.)
Ventricelli v. Kinney System Rent A Car - was not foreseeable that faulty trunk latch on a rental
car would result in a collision between two parked vehicles
** If defendant can take plaintiffs conduct and turn it into a superseding cause, then plaintiff
fails in proving element 5 and there is no apportionment of fault (as there would be if defendant
only asserted and proved an affirmative defense)

AFFIRMATIVE DEFENSES TO NEGLIGENCE CAUSE OF ACTION


a. CONTRIBUTORY/COMPARATIVE FAULT
i.
Contributory Negligence 4 of Restatement 3d (defendant has the burden of proving plaintiffs
negligence); only the plaintiff can be contributorily negligent (if one defendant caused an injury
negligently, and another defendant negligently aggravated the injury, the other defendant is not
contributorily negligent)
1. According to common law rule, this was an all-or-nothing defense; even a relatively
minor failure of the plaintiff to exercise ordinary care for her own safety would
completely bar recovery (this remained true even if the defendants negligence was
extreme)
2. This is still the rule in a minority of jurisdictions
3. Majority of jurisdictions adopted comparative fault rules to permit recovery in cases
of plaintiffs contributory negligence
ii.
Pure Comparative Fault whatever your proportion of fault, thats the portion you pay (ex. NY
statute the culpable conduct attributable to the plaintiff shall not bar recovery, but the amount
of damages otherwise recoverable shall be diminished in the proportion to plaintiffs fault)
iii.
Modified Comparative Fault
1. Greater Fault Bar plaintiff is barred from recovery if the plaintiffs fault was greater
than the defendant (ex. Wisconsin statute in cases of multiple defendants, its not
clear if the plaintiffs fault is comparative to each individual defendant or all of them
in total)
2. Equal Fault Bar plaintiff is barred if the plaintiffs fault is as great as the combined
fault of all other persons who contributed to the injury (ex. Sollin v. Wangler)
iv.
If plaintiff has proven prima facie case against all defendants, then jury considers whether
defendant has proven all the elements of affirmative defense against plaintiff (its only after that
point that the jury works out percentages for comparative fault)
v.
Difference between Comparative Fault and Joint & Several Liability
1. Comparative fault is a system by which the judgment entered assigns percentages to
the parties liable; judgment determines who owes what
2. That is different than the question of where does the plaintiff go to collect the
money to satisfy the judgment in joint & several liability, the plaintiff can go and
16

collect the money from one or more of the jointly & severally liable parties (whoever
has the money)
3. Quantitative bars for joint & several liability have to do with when you can collect
everything from one defendant; this is not the same bar as the comparative fault bar,
which has to do with whether youre liable or not; the trigger for joint & several
liability has to do with when you have to pay for the other defendant
4. Restatement 3d 10 effect of joint & several liability
vi.
Sollin v. Wangler jury determined 50/50 fault, wanting to award plaintiff half the damages
amount, but trial court dismissed the case with prejudice because this was an equal fault bar
jurisdiction; plaintiff argued on appeal that judge shouldve given Ultimate Outcome jury
instructions; jury should know what will happen based on the percentages they come up with
1. Appellate court held that trial court did not err in not giving the jury instruction
because plaintiffs counsel had not requested it had the attorney requested it, it
wouldve been proper to give the instruction
vii.
Wassell v. Adams plaintiff let man in to her motel room who then raped her; plaintiff sued
motel owners for failure to warn plaintiff about the dangerous nature of the town and failure to
take other precautions; jury found defendants 3% liable, plaintiff 97% liable; circuit court
assessed comparative fault by comparing the costs of non-negligent conduct for each party (the
cost to defendants to have a security guard on duty wouldve been much more than the cost to
the defendant to use greater vigilance in averting the attack)
1. Restatement 3d 8 Factors for assigning shares of responsibility
viii.
Mercer v. Vanderbilt University plaintiff was in a car accident (had blood-alcohol level of .20%);
while in the hospital, he was given a paralytic drug so CT scan could be done (this prevented him
from breathing on his own; at one point ventilator was not helping him breathe and his heart
stopped; he was resuscitated but suffered severe and permanent brain damage
1. Plaintiff argues for 2 separate and distinct injuries (car crash injuries and hospital
injuries); defendant argues 1 indivisible injury they alleged contributory negligence
was the but-for and proximate cause
2. Plaintiff wins the case b/c theres a rule in this jurisdiction that says health care
providers may not reduce or avoid liability for negligent treatment by asserting that
the patients injuries were originally caused by the patients own negligence
otherwise hospital could claim contributory negligence for most patients (thats why
theyre in the hospital -- some accident of their own causing)
ix.
Bexiga v. Havir Manufacturing products liability case; minor operating power press at work
had his hand crushed by the machine; court held that defense of contributory negligence was
unavailable because the negligence of the plaintiff caused the very type of injury that defendant
had a duty to protect against occurring (plaintiff cannot be contributorily negligent for an act that
caused harm if defendants duty was to guard against such harm)
1. Rule from McNamara when the defendant undertakes to protect the plaintiff from
her own fault, or when the law imposes such a duty, the entire responsibility for care
by definition falls upon the defendant and the plaintiffs fault cannot be held against
her
x.
Restatement 3d 26 apportionment of liability when damages are divisible; youre not liable
for what you did not cause, but the question of who has to prove the divisibility of the injury,
thats a question of fact and the burden of proof is on the party who wants to divide the injury
(such as in the fish kill case)
b. ASSUMPTION OF RISK
i.
Basic policy behind assumption of risk: plaintiffs should be able to agree in advance not to sue
defendants for putting them at risk (ex. If you want to go skydiving, you should be able to assume
the risk and not sue the company if something goes wrong express assumption contractual)
ii.
Restatement 3d 2 - Contractual Limitations on Liability (rule thats predicated on the existence
of a contract)
17

iii.

iv.

v.

vi.

1. express contract doesnt necessarily mean written contract (just means a valid
contract)
2. the valid contract is a complete bar to plaintiffs recovery
3. the question comes down to whether we enforce the contract; we do enforce unless
that contract is void as contrary to public policy
Moore v. Hartley Motors plaintiff was injured at an ATV rider safety class; defendants raised
affirmative defense that plaintiff had signed a release waiving her right to sue
1. plaintiff argues that contract is unenforceable because she was forced to sign in
order to participate in the class; defendants say participation was voluntary (the class
was not a requirement for purchase of the vehicle) this is not like Tunkl where
patients were forced to waive rights in order to gain essential services (if hospital
could require everyone to sign a release, they could suspend tort law and relieve
them of any liability bad public policy)
2. plaintiff then argued that if the contract will be enforced, it is outside the scope of
liability - that what the plaintiff agreed to was regular risk inherent in ATV riding, and
that the injury fell outside that scope
a. Defense argues that hitting a rock is a risk inherent to ATV riding (its an allterrain vehicle!) and that it was covered by the risk
3. Court held that this is a jury question and summary judgment shouldnt have been
granted
Implied Assumption no express agreement, written or oral
1. Its possible that even in the absence of a contract, the plaintiff did assume the risks
and waived the right to sue via plaintiffs conduct
2. Before comparative fault, assumption of risk was a complete defense now there
are three types of approaches courts take:
a. Still a complete defense (like express assumption of risk)
b. Still an affirmative defense; jury can take assumption of risk into account
when it decides percentages
c. Abolish the affirmative defense completely, or merge it with comparative
fault
Crews v. Hollenbach (Primary assumption of risk jurisdiction) defendant struck a natural gas
line and did not report it to anyone; defendant was employee of the gas company; while working
to dissipate the gas, it ignited causing plaintiff severe injuries
1. Elements of the Assumption of Risk defense:
a. Knowledge of the risk
b. Appreciated the risk
c. Voluntarily exposed himself to the risk
2. Court applies an objective standard to the first 2 elements (what a reasonable person
in Crews position would have known)
3. Some courts interpret these elements subjectively if the standard is subjective, the
case is much more likely to get to the jury (b/c plaintiff can testify, and the credibility
of the plaintiff is a jury question)
4. In this case, court held for defendant as a matter of law (no reasonable jury can
decide the 3 elements against the defendant in this case) based on the evidence
5. Restatement 523
Turcotte v. Fell plaintiff was a jockey who was thrown from his horse during a race; sued other
jockey claiming he had engaged in foul riding; Trial court granted defendants motion for
summary judgment, holding that plaintiff, by participating in the sport, relieved other
participants of any duty of reasonable care with regard to known dangers
1. Rule: there is no duty of care owed by participants of a sport; you cannot sue for
injuries that result from a risk inherent in the sport
18

a. exception = voluntary participant does not assume a risk not inherent in the
sport, such as intentional (battery) or reckless conduct (rules of the sport can
guide the fact finding, of what is and isnt inherent)
2. Plaintiff voluntary participated in an activity that he knew carried certain risks (his
participation was implied consent)

VI.

DUTIES OF LANDOWNERS
a. Traditional common law classifications of entrants on to land:
i.
Trespasser (Restatement 329) any person who has no legal right to be on anothers land and
enters the land without the express or implied consent of the owner
ii.
Licensee (Restatement 330) someone who is on the land with permission, but with a limited
license to be on the premises
iii.
Invitee (Restatement 332) any person on the premises (1) at least in part for the pecuniary
benefit of the landowner (business invitee) or (2) who is on premises held open to the general
public (public invitee)
iv.
Social Guests could be invitee or licensee (depending on the jurisdiction)
b. Gladon v. Greater Cleveland RTA - Plaintiff left a baseball game drunk, got on the subway, and got off at
the wrong stop; said he was chased and attacked by two men, and somehow ended up on track. His legs
were draped across the tracks when a train pulled into the station
i.
Plaintiffs theories of negligent conduct: (1) negligent operation of the train, and (2) lack of
security on the platform (summary judgment was granted to defendant on that claim)
ii.
Plaintiff argued that he was an invitee (because hed bought a ticket), so defendant owed him a
duty of reasonable care; defendant argued that once plaintiff entered the track area he became
either a licensee or trespasser
1. If he jumped on the tracks to flee his attackers, then hes a licensee because he had a
privilege of necessity
2. If he was pushed, then hes a trespasser (had no privilege and was not allowed to be
there)
iii.
Duties owed
1. Invitee: reasonable care
2. Licensee & Trespasser: duty varies depending upon whether or not you knew or
shouldve known they were there; prior to the time you knew they were there or
shouldve known, defendant owes plaintiff duty to refrain from intentional, wanton
or reckless conduct which is likely to injure him (affirmative activities)
iv.
So even before defendant knew or had reason to know plaintiff was on the tracks, they had an
obligation not to operate the train in a way that was reckless; once they knew he was on the
tracks (or shouldve known he was there) then there is a duty of ordinary care
v.
Rule: The duty of care owed to licensee or trespasser is to refrain from willful, wanton or reckless
conduct; upon discovery or notice of imminent danger to the entrant, landowner is required to
use ordinary care
c. Bennett v. Stanley (Duty to Child Trespasser) 5 yr old boy fell into neighbors pool and drowned;
mother drowned while trying to save him; father/husband brought suit against neighbor alleging theyd
created an unreasonable risk of harm
i.
Defendants motion for summary of judgment based on theory that decedents were trespassers,
and so the duty of care owed was to refrain from wanton or reckless or willful conduct
ii.
Plaintiff wants court to adopt a different rule for children who are trespassers; that children
trespassers are due a higher standard of care than for regular trespassers (Restatement 339)
balancing policy interests of protecting children and preserving property rights
iii.
Duty owed to the mother - the attempted rescue gives her a privilege to be on the land, which
might elevate her to licensee status (but the court says that if your privilege is rescue, you get the
same duty of care as the person youre trying to rescue)

19

d. OSullivan v. Shaw plaintiff was social guest at defendants home; plaintiff dove head first into the
shallow end of swimming pool b/c he thought he could clear it; sued defendant claiming he negligently
failed to warn of the danger
i.
Court granted summary judgment for defendant because of open and obvious danger rule:
landowner has no duty to warn of an open and obvious danger to anyone (status of the entrant
does not matter)
ii.
Open and Obvious Danger rule does not have to do with assumption of risk (Assumption of risk
focuses on the plaintiffs failure to exercise due care for his own safety, assuming that defendant
did owe plaintiff a duty of care. Open and obvious danger concerns the existence of defendants
duty of care, which the plaintiff must establish in the prima facie case)
e. Minnich v. Med-Waste (Firefighters Rule) - common law doctrine that precludes firefighters (and other
public employees, police officers) from recovering against a defendant whose negligence caused the
firefighters on-the-job injury
i.
There are many variations and exceptions to the rule (ex. you can sue arsonists); there are a lot
of borderline cases
f. Rowland v. Christian plaintiff was social guest at defendants apartment; plaintiff injured hand on a
cracked bathroom faucet that defendant was aware of
i.
Court in this case rethinks the law in a fundamental way
ii.
Instead of judges deciding what duty is owed (based on the categories of entrants), the court
eliminates the categories everyone should get the same standard of reasonable care and it
should be up to the jury to decide if the duty was breached
g. Recreational Use Statutes duty of care is owed to those who have paid to enter and those who are
expressly invited rather than merely permitted. Otherwise, liability is only for willful or malicious failure
to guard or warn against a dangerous condition
h. Lessors traditional rule is that landlord is not liable for injuries to his tenants and their guests resulting
from defects on the premises (there are exceptions). But courts have adopted new rules that landlords
are under a duty to exercise ordinary care in the maintenance of the property

VII.

DUTIES OF MEDICAL AND OTHER PROFESSIONALS


a. Restatement 299A
b. Medical Malpractice professional liability negligence cause of action with different rules with respect
to the duty of care; the other elements (3,4,5) are still part of the prima facie case
c. Standard of Care (element 1) the duty the law imposes on professionals: what you are entitled to under
the law of torts is the degree of skill that reasonably competent practitioners in the field in fact provide
under similar circumstances (that makes the customary practice of the professionals the legal standard)
i.
this goes against the usual common law rule for non-professionals (custom is not the legal
standard; it can only be used as some evidence)
ii.
Jurys job under element 2 is to take the duty of care and apply it to the circumstances (but they
have to be educated about what doctors do in those circumstances via expert testimony)
1. Jury has to determine what the standard of care among doctors is, and then
determine what it was the defendant did and whether or not what the defendant did
fell short of what is normally done (element 2)
iii.
There can be affirmative defenses in terms of the patient negligence in communicating with
the doctor, aggravating own injuries, etc.
d. Walski v. Tiesenga - plaintiff in malpractice suit must generally establish the standard of care through
expert testimony, unless common knowledge would allow layperson to recognize or infer negligence;
plaintiff in this case did not establish prima facie case just by presenting expert testimony of a differing
opinion (the testimony did not establish an accepted medical standard, only what he wouldve done in
the situation)
e. Medical standard particular custom or procedure used under very particular circumstances
f. Common jury instruction physician must possess the learning, skill and ability of other physicians, must
exercise reasonable care in the use of this knowledge and skill and must use his or her best judgment in
the care of the patient
20

i.

g.

h.

i.

j.

k.

l.

m.

Defense instructions (rejected in some cases) physician is not liable for an honest mistake or
bona fide error, or physician is not expected to be infallible or that he does not guarantee results
Judge will grant directed verdict for defendant if plaintiffs expert witness fails to establish medical
standard of care (or if plaintiff provides no expert testimony; expert is not needed only in cases of
obvious negligence, ie. amputating wrong limb)
Divided medical opinion - when competent medical authority is divided, a physician will not be held
responsible if in the exercise of his judgment he followed a court of treatment advocated by a
considerable number of recognized and respected professionals in his given area of expertise
Vergara v. Doan - court abandons the modified locality rule (since there is no longer a wide disparity in
care between urban and rural practices) and adopts a new rule: a physician must exercise that degree of
care, skill, and proficiency exercised by a reasonably careful, skillful, and prudent practitioners in the
same class to which he belongs, acting under the same or similar circumstances
i.
Strict locality rule measures conduct against that of other doctors in the same community
ii.
Modified locality rule measures conduct against that of other doctors at the time of the
operation and in similar localities
iii.
Problem with local standard only a local physician, or one who knows what the local standard is
(not just what it should be), can testify against the defendant
Hirpa v. IHC Hospitals court held that Good Samaritan law does apply to a doctor responding to an
emergency call b/c doctor had no preexisting duty to the patient and the law is meant to encourage aid
without fear of liability (Good Samaritan law states: no person who in good faith renders emergency care
at the scene of the emergency shall be liable for any civil damages as a result of any act or omission by
such person in rendering emergency care); court held that it did not matter whether the emergency was
in a hospital or on a roadside
Velazquez v. Jiminez court held that Good Samaritan law does not apply to emergency care rendered
in hospitals b/c the purpose of the law was to encourage volunteers to render aid in situations where
they would not otherwise (hospitals are supposed to render aid)
i.
HOLDING: Good Samaritan immunity encompasses only those situations in which a physician (or
other volunteer) comes, by chance, upon a victim who requires immediate emergency medical
care, at a location compromised by lack of adequate facilities, equipment, expertise, sanitation
and staff
Smith v. Knowles wrongful death suit (death of wife and stillborn baby); court held that crossexamination of defendant-doctor and excerpts from medical treatises were insufficient to establish
standard of care and causation; plaintiff offered no expert witness testimony and so defendants motion
to dismiss was upheld
i.
Expert testimony was necessary to bridge the gap between the medical texts (general info) and
the specific patient we dont have evidence of the defendants departure from the standard of
care
Res Ipsa Loquitur in malpractice cases same doctrine gets applied in the same way in medical
malpractice situations; clearest examples of medical res ipsa loquitur are cases where instruments or
sponges are left in the patient after surgery is complete, and cases where injury is inflicted on a part of
the body not being operated on (common layperson knowledge could make such inferences of
negligence without expert testimony)
i.
State v. Lourdes Hospital - court held that expert testimony could be used to help jury determine
if the injury is the type that would not have occurred in the absence of negligence
ii.
If a medical procedure carries with it the inherent risk of a particular kind of injury or
complication, the court cannot use res ipsa loquitur merely because such an injury occurred
iii.
Ybarra v. Spangard - court held that where a plaintiff receives unusual injuries while unconscious
and in the course of medical treatment, all those defendants who had any control over his body
or the instrumentalities which might have caused the injuries may properly be called upon to
meet the inference of negligence by giving an explanation of their conduct (burden shift presumption jurisdiction)
1. Policy arguments:
21

VIII.

IX.

a. if the injury doesnt ordinarily occur in the absence of negligence, in fairness


somebody should pay for the injury (it wasnt a preexisting condition)
b. The reason the plaintiff was unconscious was b/c the defendants induced
him to trust them
n. Informed Consent breach of duty of care by not informing patient of risk
i.
Harnish v. Childrens Hospital plaintiff underwent an operation to remove a tumor in her neck;
during the procedure her hypoglossal nerve was severed, allegedly resulting in a permanent and
almost total loss of tongue function; plaintiff sued doctor for failing to inform her of this risk
1. Court adopts a new rule: a physician owes to his patient the duty to disclose in a
reasonable manner all significant medical information that the physician possesses or
reasonably should possess (to be established by expert testimony) that is material to
an intelligent decision by the patient whether to undergo a proposed procedure (to
be determined by the jury) - this is the new standard of care in informed consent
cases
2. Affirmative Defense - The defendant may claim a privilege of non-disclosure if
disclosure of the risk wouldve complicated patients condition or rendered him unfit
for treatment (it is defendants burden to prove such privilege)
NONFEASANCE liability for inaction (you had a duty to do something, to step in, to be more than a
bystander but you didnt)
a. Restatement 314 - states the basic common law rule knowledge of a danger and opportunity to
prevent harm are not enough to impose duty to take action (rule stems from the notion of autonomy)
b. Circumstances When Duty Arises
i.
Restatement 314A if you have a special relationship with the victim, that creates a duty of
reasonable care (b/c of relationship youre not a mere bystander) Restatement 3d recognizes
additional relationships (pg. 492)
ii.
Restatement 321 if you created the dangerous situation, then you have a duty to act
iii.
Restatement 322 if you did something innocently but caused harm to another, then youre
under a duty to prevent further harm
iv.
Restatement 323 if you undertake rendering of services (either for pay or for free) you have a
duty of reasonable care (ex. Giles/Otis elevator case)
1. courts in lost chance malpractice suits cite this section (duty of reasonable care
was triggered b/c doctor undertook care)
c. Wakulich v. Mraz - Illinois law eliminated liability of social hosts for providing alcohol (there was no
cause of action just because defendants had offered decedent money to drink a quart of Goldschlager),
but court held that there was a cause of action because defendants had voluntarily assumed a duty of
care for decedent (when they carried her into the family room, propped her head up, etc) they were
negligent in failing to seek professional medical attention and preventing others from helping
i.
Rule: One who voluntarily undertakes to render services to another is liable for bodily harm
caused by his failure to perform such services with due care or with such competence and skill as
he possesses
d. Farwell v. Keaton - court held that defendant had a duty to render reasonable care because there was a
special relationship between him and the decedent (companions engaged in a common undertaking
social venture); this is a controversial case - did defendant have an obligation to pull him out from under
the car to begin with? If they were companions on a social venture, then he still couldve been sued
because he had a duty of care even if he hadnt begun a rescue (but the relationship wasnt custodial,
and defendant didnt put the friend in harm)
e. Where the defendant discontinues aid, the Restatement imposes liability if the defendant, by acting
unreasonably, has left the victim in a worse position (Restatement 3d pg. 491)
DUTY TO PROTECT FROM THIRD PERSONS - the general rule is that the defendant owes no duty to
protect the plaintiff from a third persons conduct (but there are exceptions)
a. DEFENDANTS RELATIONSHIP WITH THE PLAINTIFF
22

i.

ii.

iii.

iv.

Posecai v. Wal-Mart Stores woman was robbed in the Sams Club parking lot; sued store for
failing to provide security guards; Defendant argues they did not have a duty, and that it would
be unfair to require them to have security guards for the parking lot
1. Defendant may have had duty b/c plaintiff was a business invitee on their property
plaintiff argues that it should be left to the jury to decide what a reasonably prudent
person would provide for protection
2. Defendant argues that the store wasnt in a high-crime area (was adjacent to the
areas), and the type of harm claimed by plaintiff was not foreseeable
3. 4 types of foreseeability tests:
a. Specific Harm Rule a landowner does not owe a duty to protect patrons
from the violent acts of third parties unless he is aware of specific, imminent
harm about to befall him (outdated approach generally seen as too
restrictive)
b. Prior Similar Incidents Test foreseeability is established by evidence of
previous crimes on or near the premises; courts consider the nature and
extent of previous crimes, as well as their recency, frequency and similarity
to the crime in question (can lead to arbitrary results since theres no set
standard when evaluating the number of previous crimes and degree of
similarity necessary to give rise to a duty)
c. Totality of the Circumstances Test the court considers the number, nature
and location of prior similar incidents, but the lack of prior similar incidents
will not preclude a claim where the landowner knew or shouldve known
that criminal acts were foreseeable; courts consider property crimes or
minor offenses as precursors to more violent crimes (has been criticized for
being too broad) this is the rule the plaintiff wants
d. Balancing Test seeks to address the interest of both proprietors and
customers by balancing the foreseeability of harm against the burden of
imposing a duty to protect (like Learned Hand formula); the greater the
foreseeability and gravity of the harm, the greater the duty of care that will
be imposed on the business (this is the rule the court adopts in this case)
4. Policy arguments for the balancing test (for the judge to decide foreseeability rather
than the jury):
a. There might be a tendency for juries to find against the defendants because
would result in more security
b. To place burden on businesses to provide security may drive businesses from
high-crime areas
c. Places burden on stores when they are not responsible for the crime
5. HOLDING: the degree of foreseeability necessary to impose a duty to provide
security was not met (there had been only 3 prior incidents and they were not similar
to current case); the burden of imposing a duty outweighed the foreseeability
Restatement 344 a landowner who holds land open to the public for business purposes is
subject to liability to invitees for physical harm caused by third persons and by the failure of
landowner to exercise reasonable care to (a) discover that such acts are being done or are likely
to be done, or (b) give adequate warning to enable invitees to avoid the harm
Parish v. Truman held that defendant (social host) had no duty to protect plaintiff (guest)
against criminal attacks by third persons General Rule: in the absence of some special
relationship, a private person has no duty to protect another from criminal attacks by third
persons (special relationships would be carrier-passenger, innkeeper-guest, landowner-invitee,
custodian-ward Restatement 314A)
Employers Duty of Care Restatement recognizes duty of employer to protect employees from
third parties, but only if the employee comes into a position of imminent danger and this is
known to the employer
23

Schools Duty of Care to Students Marquay v. Eno 3 female students filed suit against school
employees and the school district (respondent superior)
1. HOLDING: certain school employees did owe a duty of care to the students based on
a special relationship Because children are deprived of the protection of their
parents while in school, those who take custody of the children owe them a duty of
reasonable supervision and protection from risks that are reasonably foreseeable.
The duty does not fall upon all employees; just those who have a supervisory
responsibility (teachers, principals, and those in a position to hire and fire)
2. The criminal statute did not apply in this case:
a. By looking to legislative intent, court reasons that the statute does not
support a private right of action
b. The statute also cant be used to establish a duty of care because the duty
the statute speaks of is a reporting duty, which is different than the duty at
issue (duty of supervision)
3. Mirand v. City of New York a student was attacked by other students; court held in
favor of the student stating that schools are under a duty to supervise students in
their charge and will be liable for foreseeable injuries proximately related to the
absence of adequate supervision (the duty derives from the fact that schools, in
assuming physical custody of students, take the place of parents); the care owed is
that of a parent of ordinary prudence in comparable circumstances
4. Young v. Salt Lake City Sch. Dist. court held school not liable for injuries sustained
by student on bicycle on his way to an after-school meeting; when a school lacks
custody, it has no protective obligation and no special relationship exists
b. DEFENDANTS RELATIONSHIP WITH DANGEROUS PERSONS
i.
Duty of Parent to control conduct of Child basic rule: parents are not responsible for the
actions of their children, but under Restatement 316, parents are not mere bystanders; parent
can be liable for failing to control some specific dangerous habit of a child of which the parent
knows or should know in the exercise of reasonable care
1. Plaintiff must show that parent had reason to know with some specificity of a
present opportunity and need to restrain the child to prevent some imminently
foreseeable harm
ii.
Tarasoff v. Regents of U of CA patient confessed to his therapist his intent to kill a girl;
therapist had the patient detained by campus police, but he was later let go; patient ended up
killing the girl; girls parents brought suit against therapist, the supervisor, the campus police and
the university
1. Theory of negligent conduct: failure to warn of impending danger
2. Defendants argue that they owed no duty of care, that they were mere bystanders
knowledge of the threat and an opportunity to do something about it were not
enough to trigger a duty
3. FORESEEABILITY OF HARM IS NOT ENOUGH TO CREATE A DUTY (ex. baby on the
railroad case)
4. Plaintiff argues that the special patient-therapist relationship does create a duty
5. Restatement 315
6. RULE: If defendant knew or should have known (professional standard) that patient
presented a danger, then defendant bears a duty of reasonable care (reasonable
person standard) to protect the foreseeable victim of that danger
a. The professional standard applies to diagnosis of the situation, and then
reasonable person standard applies to what the therapist should do once the
danger is recognized
7. Policy Argument:
v.

24

X.

a. To require therapists to disclose information conveyed in confidence during


therapy would render the relationship ineffective (because patients could
not be honest)
i. But court holds that public interest for safety outweighs the
individual privilege of privacy
iii.
Doctrine of Negligent Entrustment - liability results when an owner, having knowledge of a
persons incompetency, inexperience, or recklessness, entrusts his automobile or other
dangerous item to another with permission to use it owner must have actual knowledge of the
other persons incompetence
1. These are not really bystander cases because defendant helped create the risk
iv.
Brigance v. Velvet Dove Restaurant - court held that vendor of liquor for on-site consumption
has a duty to exercise reasonable care not to sell liquor to a noticeably intoxicated person,
because doing so creates a foreseeable and unreasonable risk of harm to others
1. Courts traditionally (and some still do) denied liability in these types of cases because
the alcohol provider was not the proximate cause of harm done by the drinker (the
person voluntarily drank the alcohol)
2. Restatement 308 & 390 (not in restatement book pg. 555 of textbook)
EMOTIONAL HARM - when the emotional harm is the primary compensable injury absent a bodily harm
that occurs first; can involve an intentional or unintentional tort
a. 3 kinds of emotional harm claims for compensable injury:
i.
Restatement 46 intentional or reckless (acting in disregard of high probability that emotional
distress will result) infliction of emotional harm
ii.
Restatement 436 negligent infliction of emotional harm (negligence cause of action)
iii.
Loss of Consortium
b. INTENTIONAL INFLICTION OF EMOTIONAL HARM
i.
GTE Southwest v. Bruce plaintiffs (employees) alleged emotional harm caused by outrageous
behavior of supervisor (screaming, lunging at employees, pounding his fists ongoing pattern of
abuse)
1. Elements of Cause of Action:
a. Defendant must have acted intentionally or recklessly
b. Conduct was extreme and outrageous
c. Resulting distress was severe
d. Actions of defendant caused plaintiffs emotional distress
2. RULE: the emotional distress must be the intended or primary consequence of the
defendants conduct (cannot be incidental to the commission of some other tort)
3. Defendant argued insufficient evidence of element 2; said ; the behavior was just his
management style, and argued that if court allows that to qualify as outrageous it
will open the floodgates to litigation (because its part of supervisors responsibility
to manage, demote, discipline employees)
4. Plaintiffs argued that the regularity of the behavior went beyond the bounds of
tolerable workplace conduct (evidence in this case is sufficient for a jury to find the
conduct outrageous even under the conservative standard) court agrees and
upholds jury verdict (denying defendants motion for JNOV)
ii.
Taylor v. Metzger the conduct must be so outrageous in character, and so extreme in degree,
as to go beyond all possible bounds of decency, and to be regarded as atrocious, and utterly
intolerable in a civilized community
1. In determining whether conduct is outrageous, courts have emphasized that the
conduct (a) is repeated or carried out over a period of time, or (b) is an abuse of
power on the one hand or abuse of a person known to be especially vulnerable
iii.
Homer v. Long therapist seduced plaintiffs wife
1. Subjective standard: the defendants conduct must not only be outrageous from the
perspective of society at large, but must also be outrageous to the plaintiff
25

2. Court upholds the presence rule (in Restatement)


a. Presence Rule: where conduct is directed at one person but it outrageous to
another, the other person must be present to witness the conduct in order
to recover
3. Intentional Infliction of Emotional Distress (when physical harm is done to a third
person):
a. The plaintiff must be present when the injury occurred to the other person
b. The plaintiff was a close relative of the injured person
c. The defendant knew that the plaintiff was present and a close relative of the
injured person
d. (the plaintiff may not have to establish presence or family relationship if he
can show that the defendant had a design or purpose to cause severe
distress to the plaintiff)
c. NEGLIGENT INFLICTION OF EMOTIONAL DISTRESS
i.
Courts want to set guidelines for what counts as severe emotional distress to prevent a flood of
litigation
ii.
Evidence of the severity of harm:
1. Impact Rule Mitchell established rule that some kind of physical impact preceding
any emotional harm is necessary (most states have done away with this rule)
2. Physical Manifestation / Objective Symptom Rule in cases where there was no
impact, the plaintiff can recover only if she produces evidence of physical harm
resulting from the shock, or some kind of physical manifestation of the shock
3. Some courts only require objective corroboration of the distress, not necessarily
physical manifestation (ex. if family watches house get blown up the nature of the
event itself corroborates the distress)
4. Medically Diagnosable Rule emotional injury must be medically diagnosable as an
emotional disorder, but not necessarily reflected in observable physical
manifestations
iii.
If negligent conduct aggravates a pre-existing condition, the victim must be compensated for the
full extent of the aggravation
iv.
Grube v. Union Pacific RR plaintiff was engineer of train that collided with a car stuck on the
tracks; plaintiff sued his employer claiming negligent infliction of emotional distress; he claimed
seeing the driver of the car before impact and then in attempting to render aid caused him
distress (became physically ill); he did not suffer any physical injury from the collision
1. Zone of Danger Test: plaintiff must be (1) within the zone of danger and (2) suffer
imminent apprehension of physical harm which causes or contributes to the
emotional injury
2. Court held that plaintiff did not satisfy this test because he did not at any time fear
for his own safety
v.
Close Relatives Outside the Zone of Danger
1. Dillon Factors court permitted recovery for emotional harm by persons who were
merely bystanders (not in the zone of danger); court held that defendant might owe
duty to protect those who might foreseeably suffer emotional harm because of the
injury to the third person Factors for determining Foreseeability:
a. Whether plaintiff was located near the scene of the accident
b. Whether the shock resulted from a direct emotional impact upon the
plaintiff from the sensory and contemporaneous observance of the accident
(as opposed to learning about it after the fact)
c. Whether plaintiff and the victim were closely related
2. Thing Conditions plaintiff may recover damages for emotional distress caused by
observing the negligently inflicted injury of a third person if and only if:
26

vi.

vii.

viii.

ix.
x.

xi.

a. Plaintiff is closely related to the victim (closely related by blood or marriage;


relatives residing in the same house; parents, siblings, children and
grandchildren of the victim)
b. Plaintiff is present at the scene of the injury producing event at the time it
occurs (and is aware that it is causing injury)
c. As a result, plaintiff suffers serious emotional distress (a reaction beyond
that which would be anticipated from a disinterested witness and which is
not an abnormal response to the circumstances)
Professional Malpractice Types of Cases
1. Burgess v. Superior Court court held that mother whose baby was injured during
delivery could not sue as a bystander (in such cases, Thing rules do not apply); there
was a preexisting relationship and a duty owed did not matter that the mother was
aware of the injury as it occurred
Loss of Consortium not only loss of support or services, also includes loss of love,
companionship, affection, society, sexual relations, solace, etc. injury to the relationship
1. Boucher v. Dixie Medical Center 18 yr old son came in to the hospital with an
injured hand; during surgery he lapsed into a coma; became severely brain-damaged
quadriplegic; parents sued for NIED and loss of consortium
a. Plaintiffs could not recover for NIED because they were not in the zone of
danger (Thing conditions also bar recovery)
b. Court held that parents could not recover for loss of consortium with adult
child (Utah had already abolished claim for loss of consortium between
spouses wouldnt make sense to then extend the claim for adult children)
c. The parents cannot claim expenses for sons nursing care b/c he could
recover such money in his own med-mal claim (court wants to prevent
double recovery)
Derivative cause of action:
1. Loss of consortium (some argue that this is an independent claim)
2. Wrongful death
3. Dillon cases observing injury to family member
4. Derivative means - any affirmative defense that would work against the victim would
work against the plaintiff; ex. Wrongful death claim - one spouse dies, the other sues
- any affirmative defense that the defendant couldve brought against the victim is
good against the plaintiff; the plaintiffs are not the direct victims
Death-related cases ex. mishandling of dead body, negligent communication that someone had
died when they didnt; courts thought they didnt need any more evidence of genuineness
Sacco v. High Country the case does not fit any previous categories for recovery under
emotional distress claim (no zone of danger, no Dillon, no medmal, no death); court relies on the
severity issue; court in this case is willing to go as far as courts have gone to realize emotional
distress as its own cause of action; court adopts this standard:
1. A cause of action for NIED will arise under circumstances where serious or severe
emotional distress to the plaintiff was the reasonably foreseeable consequence of
the defendants negligent act or omission
Potter v. Firestone Tire plaintiffs sued for emotional distress brought on by fear of cancer
caused by toxic contamination of drinking water (but there is no claim of physical injury)
1. Court is concerned about extending the emotional distress cause of action to such
cases because were all exposed to carcinogens the flood of litigation would be
endless
2. RULE: in the absence of physical injury or illness, damages for fear of cancer may be
recovered only if plaintiff proves: (1) defendants breach of duty exposed plaintiff to
carcinogens, and (2) plaintiffs fear stems from a knowledge, corroborated by
scientific evidence, that it is more likely than not that the plaintiff will develop cancer
27

3. Exception: plaintiff does not have to prove more likely than not threshold if the
defendants conduct in causing the exposure was fraudulent, oppressive or malicious
4. Policy arguments for the rule
a. Unlimited fear of cancer claims would affect availability of affordable liability
insurance for companies that deal with toxins
b. Detrimental impact on healthcare field
c. To allow compensation in such cases might make defendants and their
insurers unable to compensate victims who actually develop cancer (theyd
be bankrupt)

XI.

VICARIOUS LIABILITY
a. Respondent Superior - Employers can be held liable for the torts of certain employees, provided those
torts are committed within the scope of employment
i.
Basis for the rule: the desire to include in the cost of operation inevitable losses to third persons
incident to carrying on an enterprise, and thus distribute the burden among those benefitted by
the enterprise
b. Test for within scope of employment: whether the act was done while the servant was doing his
masters work, no matter how irregularly, or with what disregard of instructions
i.
Employers are generally not liable for the intentional torts of their employees
c. Employees usually dont have the money to compensate the injured party on the other hand,
employers can spread the impact of such operation costs via cost accounting (factoring the cost of such
litigation into the price of the product) and insurance coverage (cost is distributed among other like
insured businesses)
d. Master-Servant Relationship
i.
can be established without payment or promise of payment (ie. Servant may work for free)
ii.
Relationship is not established unless the servant submits himself to the control of the employer
iii.
Employer may be liable for voluntary work of employee if employee was still being paid while
performing the voluntary job and received some benefit from the voluntary job (such as it
voluntary work is taken into account in salary reviews); the fact that voluntary work was not
compulsory or paid extra does not relieve employer of liability
iv.
General Rule: employer is not vicariously liable for the negligence of an independent
contractor
1. Exception: employer will be liable if independent contractor was hired for a nondelegable duty of the employer; employer may delegate the work to others, but not
the duty
e. Non-delegable duties:
i.
Inherently dangerous work when it creates a peculiar risk of harm to others unless special
precautions are taken
ii.
Restatement 424: one who is required by specific statute to provide safety protections for
others may not avoid ultimate responsibility for this statutory duty by use of an independent
contractor
iii.
Often comes up in res ipsa loquitur under condition 2 (exclusive control) ex. If owner of a plane
tries to deny liability b/c they contract out the maintenance of the plane
iv.
Restatement 425: imposes liability upon one who employs an independent contractor to
maintain the safety of land held open to the public as a place of business, or a chattel supplied to
others for business uses

XII.

PRODUCTS LIABILITY
a. Types of cause of action:
i.
Contract/UCC Breach of Warranty cause of action (the product delivered is not the product you
contracted for)
ii.
Negligence cause of action
iii.
Strict Liability cause of action
1. Restatement 2d 402A 1965
28

b.

c.

d.

e.

f.

2. Case Law
3. Restatement 3d 1997
Three Kinds of Defects (you have to prove the product was defective when it was sold - at the time it left
the defendants hands):
i.
Manufacturing/Production Defect when the product you got had a defect (not a defect in the
whole product line) can occur even if there is nothing wrong with the products design
1. Definition of defect changes between the 2d and 3d Restatements
ii.
Design Defect (products generally, food, pharmaceuticals)
1. Restatement 2d Reasonable Consumer Expectations Test - comparing the
products features regarding safety to what a reasonable consumer would expect
(comment i, page 86); just because a product is dangerous doesnt mean its
defective
2. Restatement 3d Risk-Utility Test
iii.
Information Defect (warnings, directions, etc.)
Lee v. Crookston Coca-Cola (Manufacturing Defect) soda bottle exploded in waitress hand; plaintiff
wanted to present claim as strict liability, not as a negligence claim (if it was a negligence claim, it would
be a res ipsa case because plaintiff cant know for sure what defendants negligent conduct was
i.
Elements of Prima Facie case plaintiff must present evidence that:
1. The product was in fact in a defective condition, unreasonably dangerous for its
intended use
2. Such defect existed when the product left defendants control, and
3. The defect was the proximate cause of the injury
ii.
Under strict liability, plaintiff doesnt have to prove that defendant was negligent at all -- plaintiff
is not required to prove specifically what defect caused the incident, but can rely on
circumstantial evidence from which it can reasonably be inferred that it is more probable than
not that the product was defective when it left defendants control
Manufacturing Defect
i.
Consumer Expectation Test (Restatement 2d, comment i) test for determining
defect/unreasonable danger; asks whether the product was dangerous beyond the
contemplation of the consumer
ii.
Under Restatement 3d a product contains a manufacturing defect when the product departs
from its intended design even though all possible care was exercised in the preparation and
marketing of the product ( 2a) except for food (Restatement 3d retained consumer expectation
test, section 3)
Inference of Product Defect Restatement 3d 3
i.
An inference of defect may arise even when the plaintiff fails to prove what aspect of the product
was defective
Knitz v. Minster Machine Co (Design Defect) plaintiff was operating die-press when his fingers were
amputated
i.
Theory of design defect: Lack of physical guard to prevent hands from being in the way while the
press is being operated (this is just dealing with one element of the prima facie case plaintiff
must still prove that defect was a cause in fact and proximate cause of the injury)
ii.
Defendant argues that no reasonable jury could find that the feature caused the machine to fall
below consumer expectations (the machine was just as dangerous as everyone thought it was
and so its not defective)
iii.
Plaintiff argues for a change in the rule, for a new definition of defective that weighs the risks
and benefits of the design feature; design is defective if the benefits of the challenged design do
not outweigh the risk inherent in such design court adopts this Risk Utility Test (this is the
definition found in Restatement 3d)
iv.
Relevant Factors for Risk Utility Balancing:
1. Likelihood that product will cause injury
2. Gravity of the harm if it occurs
29

3. Mechanical and economic feasibility of an improved design (cost of preventing harm


by using a different design)
g. Design Defect
i.
Restatement 2d product is defective in design if the plaintiff demonstrates that the product
failed to perform as safely as an ordinary consumer would expect when used in an intended or
reasonably foreseeable manner
ii.
Restatement 3d Risk Utility Test from Knitz
iii.
Minority Rule Barker v. Lull Engineering - shifts burden of persuasion to defendant to prove
that the benefits of the design outweigh the risks once the plaintiff proves the design caused the
injury (only a few courts have adopted this approach)
h. Honda of America v. Norman car rolled backwards into lake and woman drowned b/c unable to take
off seatbelt; plaintiffs were not able to recover because failed to prove design defect of seatbelt system
i.
To prove design defect, plaintiffs had to show:
1. That there was a safer alternative
2. The safer alternative would have prevented or reduced the risk without impairing
the products utility, and
3. The safer alternative was both technologically and economically feasible
i. Design defect cases have become very expensive to litigate (strict liability was originally intended to
make these trials less costly than if plaintiff had to prove negligence, but it hasnt worked out that way);
its not just bad lawyering, it has to do with the expense of getting expert testimony that second-guesses
defendants engineers
j. Restatement 3d 2 comment d requires that the plaintiff prove that a reasonable alternative design
was available at the time the product was sold; and they must also show that the product was
unreasonably dangerous and harm that plaintiff suffered was foreseeable
k. McCarthy v. Olin Group LIRR shooting case; plaintiffs sued manufacturer of bullets for the enhanced
injuries caused by the design features of the bullet (thats the alleged defect); plaintiffs were denied
recovery
i.
Plaintiffs theory of defective design:
1. Design and manufacture of bullets were inherently dangerous
a. Defense: the bullets were just as dangerous as expected (ex. knives arent
defective b/c theyre sharp); the bullets performed exactly as advertised are
not more dangerous than the ordinary consumer expects
2. Risk posed by the bullets outweighs their utility
a. Defense: this isnt a risk, this is the benefit its designed to be harmful, and
risk-utility balancing is not for this type of product
3. Negligent marketing of the bullets to the general public
a. Defense: its not the manufacturers duty to control the distribution of such
products
l. Liriano v. Hobart Corp (Warning/Information Defect) plaintiffs was severely injured when hand was
caught in the meat grinder; he sued manufacturer of the machine; defendant filed cross-claim against
employer for failure to warn and for removing the safety guard (sues for contribution)
i.
Open and obvious dangers do not require a warning (consistent with negligence cause of action
there is no duty to warn about an open and obvious danger; ex. diving into pool)
1. But warnings do more than just tell you about dangers you already know about;
warnings tell you of ways around the danger (give information)
ii.
Court held that a reasonable jury couldve found that was a defect (not warning to inform of
safer alternatives)
m. Causation in Warning Failure cases: cant really know that but-for the lack of a warning the plaintiff
would not have been injured; to solve this problem, many courts presume that if warning had been
given, plaintiff wouldve followed it heeding presumption

30

Vous aimerez peut-être aussi